WBR0990: Difference between revisions

Jump to navigation Jump to search
No edit summary
No edit summary
Line 33: Line 33:
|Explanation=In this question we are asked to calculate the pulmonary vascular resistance for this patient.  Recall that the resistance of a vessel can be calculated by the change in pressure across the segment divided by the flow (R=P/Q).  Generally, the pulmonary capillary wedge pressure is a good estimate of the pressure in the left atrium.  Therefore, pulmonary vascular resistance is given by the formula:
|Explanation=In this question we are asked to calculate the pulmonary vascular resistance for this patient.  Recall that the resistance of a vessel can be calculated by the change in pressure across the segment divided by the flow (R=P/Q).  Generally, the pulmonary capillary wedge pressure is a good estimate of the pressure in the left atrium.  Therefore, pulmonary vascular resistance is given by the formula:


<math>PVR = (P_{pulmonary artery} - P_{capillary wedge})/Cardiac Output</math>
[[File:PVREqn101814.png]]
 
{{math|<VAR>&alpha;</VAR>}}


Plugging in values from the above case we get the following:
Plugging in values from the above case we get the following:
Line 42: Line 40:


Normal values for pulmonary vascular resistance range from 0.25–1.6 mmHg·min/L.
Normal values for pulmonary vascular resistance range from 0.25–1.6 mmHg·min/L.
'''Educational objective:''' 
'''References:''' 
|AnswerA=0.72 mmHg·min/L
|AnswerA=0.72 mmHg·min/L
|AnswerAExp=Incorrect:  See calculation in the explanation
|AnswerAExp=See calculation in the explanation
|AnswerB=1.39 mmHg·min/L
|AnswerB=1.39 mmHg·min/L
|AnswerBExp=Correct:  See calculation in the explanation
|AnswerBExp=See calculation in the explanation
|AnswerC=11.25 mmHg·min/L
|AnswerC=11.25 mmHg·min/L
|AnswerCExp=Incorrect:  See calculation in the explanation
|AnswerCExp=See calculation in the explanation
|AnswerD=13.9 mmHg·min/L
|AnswerD=13.9 mmHg·min/L
|AnswerDExp=Incorrect:  This result can be achieved by dividing the difference in systolic and diastolic aortic pressures and dividing by the cardiac output (3.6L/min)
|AnswerDExp=This result can be achieved by dividing the difference in systolic and diastolic aortic pressures and dividing by the cardiac output (3.6L/min)
|AnswerE=20.83 mmHg·min/L
|AnswerE=20.83 mmHg·min/L
|AnswerEExp=Incorrect: See calculation in the explanation
|AnswerEExp=See calculation in the explanation
|EducationalObjectives=Pulmonary vascular resistance is given by the change in pressure across the pulmonary vasculature divided by the cardiac output.
|EducationalObjectives=Pulmonary vascular resistance is given by the change in pressure across the pulmonary vasculature divided by the cardiac output.
|References=First Aid 2014 page 599
|References=First Aid 2014 page 599

Revision as of 00:20, 20 October 2014

 
Author PageAuthor::William J Gibson
Exam Type ExamType::USMLE Step 1
Main Category MainCategory::Physiology
Sub Category SubCategory::Pulmonology
Prompt [[Prompt::An attending in the cardiac intensive care unit at major academic medical center is teaching a resident about the pulmonary vasculature. A patient with a history of hypertrophic cardiomyopathy is recovering from an anterior myocardial infarction and the following values are measured.

Pulmonary artery pressure (mean): 15 Pulmonary capillary wedge pressure (mean): 10 Aortic systolic pressure: 140 Aortic diastolic pressure: 90 Heart rate: 90 per minute Stroke Volume: 40 mL

Which of the following corresponds to the pulmonary vascular resistance in this patient?]]

Answer A AnswerA::0.72 mmHg·min/L
Answer A Explanation AnswerAExp::See calculation in the explanation
Answer B AnswerB::1.39 mmHg·min/L
Answer B Explanation AnswerBExp::See calculation in the explanation
Answer C AnswerC::11.25 mmHg·min/L
Answer C Explanation AnswerCExp::See calculation in the explanation
Answer D AnswerD::13.9 mmHg·min/L
Answer D Explanation AnswerDExp::This result can be achieved by dividing the difference in systolic and diastolic aortic pressures and dividing by the cardiac output (3.6L/min)
Answer E AnswerE::20.83 mmHg·min/L
Answer E Explanation AnswerEExp::See calculation in the explanation
Right Answer RightAnswer::B
Explanation [[Explanation::In this question we are asked to calculate the pulmonary vascular resistance for this patient. Recall that the resistance of a vessel can be calculated by the change in pressure across the segment divided by the flow (R=P/Q). Generally, the pulmonary capillary wedge pressure is a good estimate of the pressure in the left atrium. Therefore, pulmonary vascular resistance is given by the formula:

Plugging in values from the above case we get the following: (15 mmHg - 10 mmHg)/(90 beats/minute*(.040 L/min))=5/3.60=1.39 mmHg·min/L.


Normal values for pulmonary vascular resistance range from 0.25–1.6 mmHg·min/L.
Educational Objective: Pulmonary vascular resistance is given by the change in pressure across the pulmonary vasculature divided by the cardiac output.
References: First Aid 2014 page 599]]

Approved Approved::Yes
Keyword WBRKeyword::Lung, WBRKeyword::Vasculature, WBRKeyword::Pulmonary hypertension, WBRKeyword::Hemodynamics, WBRKeyword::Hemodynamic
Linked Question Linked::
Order in Linked Questions LinkedOrder::